Patho Midterm Study

Réussis tes devoirs et examens dès maintenant avec Quizwiz!

A student makes the statement to a colleague, "Blood plasma is essentially just a carrier for the formed cells like red blood cells and white blood cells." What would be the most accurate response to this statement?

"Actually, plasma plays a significant role in nutrient and waste transport."

A client diagnosed with H. pylori asks the nurse, "How can an infection occur in the stomach since it is an acid environment?" The nurse responds:

"H. pylori produces an enzyme called urease that converts gastric juices into ammonia, which neutralizes the acidic stomach environment."

A 60-year-old male client with an acute viral infection is receiving interferon therapy. The clinician is teaching the client about the ways interferon differs from other anti-infective therapies. What point should the clinician include?

"Interferon stimulates the activity of natural killer cells that attack viruses."

Which statement listed below is accurate regarding the functions and nature of cytokines relative to a variety of pathologies?

"A particular cytokine can have varied effects on different systems, a fact which limits their therapeutic use."

A hospital client has a large, superficial wound on her elbow that was the result of shearing action when she was moved up in her bed. The client's husband mentions that the wound looks infected and irritated since it is completely red. Which response by the nurse would be inappropriate?

"A thin sheet of blood clotting is actually desirable and not a sign that your wife's wound is infected."

During a flu shot clinic, one of the questions asked relates to whether the client has a history of Guillain-Barré syndrome. The client asks, "What is that?" How should the clinician reply?

"A type of paralysis that affects movement on both sides of the body. It may even involve the respiratory muscles."

Which student statement demonstrates a sound understanding of the cellular processes of hypertrophy and hyperplasia?

"I know that cells like neurons have little capacity for hyperplastic growth."

A community health nurse is conducting the nutritional component of a class for new mothers. Which teaching point would be most justified?

"If you choose to feed your baby with formula, ensure that it is iron-fortified."

The daughter of a client with metastatic cancer is confused as to why the client is receiving radiation therapy and asks the nurse, "Why are we still treating the cancer? The plan was only for comfort care." What is the nurse's best response?

"In this case, the radiation is being used to help alleviate the bone pain your family member is experiencing."

Which statement by a client who has a new diagnosis of non-Hodgkin lymphoma (NHL) demonstrates a sound understanding of the diagnosis and treatment of the health problem?

"They confirmed my diagnosis with a lymph node biopsy and I'll get radiation treatment soon because it's fairly early stage."

The nurse is preparing to administer rasburicase to a client undergoing chemotherapy. How should the nurse explain the purpose of this medication to the client?

"This drug helps reduce uric acid levels and protect your kidneys."

A 36-year old woman with a diagnosis of antiphospholipid syndrome is receiving a scheduled checkup from her nurse practitioner. Which teaching point would the nurse most likely prioritize?

"We need to ensure your birth control pills don't contain estrogen."

A 20 year-old has been diagnosed with an astrocytic brain tumor located in the brainstem. Which statement by the oncologist treating the client is most accurate?

"Your prognosis will depend on whether we can surgically resect your tumor."

A surgeon is explaining to the parents of a 6-year-old boy the rationale for the suggestion of removing the boy's spleen. Which teaching point would be most accurate?

"Your son's spleen is holding on to too many of his platelets so they're not available for clotting."

The nurse is counseling a client who is going for major surgery and must stop taking aspirin to reduce the risk for bleeding. The nurse knows aspirin decreases platelet activity for the duration of the platelet's lifespan. How many days prior to the surgery should the nurse tell the client to stop taking aspirin?

10

During a period of extreme excess fluid volume, a renal dialysis client may be administered which type of IV solution to shrink the swollen cells by pulling water out of the cell?

3% sodium chloride.

Which client would be considered to be in the latent period of HIV infection?

33-year-old heroin drug abuser who has numerous enlarged lymph nodes in his axilla and cervical neck region for the past 4 months.

Which client has an absolute neutrophil count (ANC) that is critically low and the standard of care would recommend placement on neutropenic precautions?

37-year-old client with leukemia being treated with chemotherapy with ANC of 400 (0.40 x 109/L).

When talking about the lifespan of various blood components, the students should know that once a neutrophil moves into tissue, it lives approximately how long?

4 days.

A fifth-grade elementary student asks the school nurse how much blood is in an entire body. The nurse should respond that the average grown-up adult has:

5 to 6 liters of blood throughout the body

Which client would have a very poor response related to tissue regeneration of his or her injured area?

54-year-old male who had a massive MI 4 days ago and came to the ED today for treatment.

Which client may be experiencing a sensory focal seizure that has sent an abnormal cortical discharge to the autonomic nervous system (ANS)?

56-year-old reporting tingling sensations and has both an elevated pulse and BPA

A medical student is familiarizing herself with recent overnight admissions to an acute medical unit of a university hospital. Which client would the student recognize as least likely to have a diagnosis of antiphospholipid syndrome in his or her medical history?

A 21-year-old male with a diagnosis of cellulitis and suspected endocarditis secondary to intravenous drug use

In which client would diagnostic investigations be least likely to reveal increased thrombopoietin production?

A 21-year-old woman awaiting bone marrow transplant for myelogenous leukemia

The nurse is planning care for a group of clients. Which client should the nurse assess first?

A 25-year-old client with an absolute neutrophil count of 300/mm3 (0.30 x 109/L)

Which client of a primary care physician would not require extra screening for cancer?

A 38-year-old female with Down syndrome and congenital scoliosis

In which individuals would a clinician most suspect multiple myeloma as a diagnosis?

A 40-year-old man who has had 3 broken bones over the past 6 months and whose serum calcium and creatinine levels are elevated.

The nurse is caring for a group of postoperative clients. Who should the nurse prioritize as having the greatest risk for infection at the surgical site?

A 45-year-old with external fixation of a fractured tibia

Which situation can best be characterized as an example of passive immunity?

A 6-week-old infant receives antibodies from his mother's breast milk.

Which client is most likely to be a candidate for a thymus transplant as the treatment of choice to reconstitute T-cell immunity?

A 7-year-old boy with diagnosis of thymic aplasia whose blood work indicates absence of T cells

Which client would be most likely to be experiencing an increase in renal erythropoietin production?

A 71-year-old smoker admitted to hospital with exacerbation of his chronic obstructive pulmonary disease (COPD)

Which individuals would most likely possess normal plasma cell synthesis and fully differentiated myeloid and lymphoid cells?

A 78-year-old male who has been diagnosed with chronic lymphocytic leukemia (CLL).

Which individual situation listed below best exemplifies the processes of innate immunity?

A child who has experienced heat and swelling of his skinned knuckle

Which scenario best describes an example of infection originating with a fomite?

A client who contracted hepatitis C through sharing a contaminated syringe with an infected person

A 22-year-old female college student is shocked to receive a diagnosis of myasthenia gravis. What are the etiology and most likely treatment for her health problem?

A decline in functioning acetylcholine receptors; treatment with corticosteroids and intravenous immunoglobulins

A client with a diagnosis of atrophic gastritis and consequent pernicious anemia is receiving high oral doses of vitamin B12. Following completion of his treatment, which lab finding demonstrates improvement in this condition?

A decrease in mean corpuscular volume

Which individual would be most likely to experience global ischemia to his or her brain?

A man who has entered cardiogenic shock following a severe myocardial infarction

The Cardiovascular Technologist evaluates the ECG of a client who has developed hypokalemia secondary to diuretic use. Which manifestation of hypokalemia does the clinician anticipate on the ECG?

A prominent U wave and a flattened T wave.

A client is experiencing muscle atrophy following 2 weeks in traction after a motor vehicle accident. Which factor has most likely contributed to the atrophy of the client's muscle cells?

A reduction of skeletal muscle use secondary to the traction treatment

A middle school student is scheduled to receive booster immunizations and the father asks why the booster is necessary. What characteristic of the adaptive immune system listed below would provide the rationale for the clinician's response?

A secondary response causes a sharp rise in antibody levels.

A 24-year-old woman presents with fever and painful, swollen cervical lymph nodes. Her blood work indicates neutrophilia with a shift to the left. She most likely has:

A severe bacterial infection

A 26 year-old female is resting after a 1-minute episode during which she lost consciousness while her muscles contracted and extremities extended. This was followed by rhythmic contraction and relaxation of her extremities. On regaining consciousness, she found herself to have been incontinent of urine. What has the woman most likely experienced?

A tonic-clonic seizure

A client with a gastrointestinal bleed secondary to alcohol abuse and a hemoglobin level of 5.8 g/dL (58 g/L) has been prescribed a transfusion of packed red blood cells. The client possesses type B antibodies but lacks type D antigens on his red cells. Transfusion of which blood type would be least likely to produce a transfusion reaction?

A-

A 32-year-old woman presents at her health clinic complaining of weakness, feeling of abdominal fullness, 6-month history of fatigue and night sweats. She added a multivitamin with iron and some extra meat and leafy greens to her diet but has not experienced an increase in energy. Upon assessment, her spleen was noted to be enlarged. Which diagnosis is most likely associated with her manifestations?

Accelerated chronic myelogenous leukemia

Which phenomenon would be least likely to result in activation of the complement system?

Activation of toll-like receptors (TLRs) on complement proteins.

The nurse is planning care for a 6-hour-old neonate who has been born with cleft palate. What aspect of care should the nurse prioritize?

Adequate nutrition

The nurse is caring for a client who meets the physical assessment criteria for sepsis, but the specimen cultures are negative for a causative organism. What action should the nurse take next?

Administer the prescribed antibiotics.

A 72-year-old client being treated for rheumatoid arthritis is admitted with suspected ehrlichiosis. What should the nurse implement as a treatment priority?

Administration of empirically chosen antibiotics

A 45-year-old woman with a diagnosis of shingles is experiencing an acute onset of severe neuropathic pain. Which stage of Selye's characterization of stress response is the woman most likely experiencing at the moment?

Alarm

An 81-year-old female client in a subacute medical unit of a hospital has developed an oral Candida albicans infection. Which phenomena would the client's nurse suspect as a key contributing factor to her infection?

Antibiotic therapy that eliminated normal bacterial flora

A client is having an edrophonium test. The client suddenly goes into respiratory arrest. Which priority medication should the nurse be prepared to administer?

Anticholinergic

A client is admitted for cardioembolic stroke. Which therapy to best prevent recurrence of embolic stroke should the nurse monitor for effectiveness?

Anticoagulation therapy

Following a spider bite she received while camping, a 20-year-old female presented to the emergency department with rash, edema, and fever and was subsequently diagnosed with serum sickness. Which statement best conveys the physiologic rationale for the broad systemic effects of this event?

Antigen-antibody complexes have been deposited in a variety of locations throughout the body.

A client with a surgical wound has developed excessive granulation tissue extending above the wound edges. Which action should the nurse take?

Ask the health care provider about surgical options to remove the excess tissue and promote wound healing.

The clinician is caring for a client hospitalized yesterday with acute coronary syndrome. The client also has difficulty dealing with life stressors. What should be focus as the priority of care?

Assessing heart rate and blood pressure and interviewing the client about chest pain

A 47-year-old woman was diagnosed with amyotrophic lateral sclerosis 3 years ago and has experienced a progressive onset and severity of complications. She has been admitted to a palliative care unit due to her poor prognosis. What assessments and interventions should the clinical staff of the unit prioritize in their care?

Assessment of swallowing ability and respiratory status.

Which statement most accurately conveys an aspect of lymphatic system activity?

B and T lymphocyte development begins in the bone marrow and ends in the peripheral lymphoid structures.

Following a bone marrow biopsy, which assessment would indicate the client is experiencing a complication as a result of this diagnostic procedure?

BP 90/60, heart rate 132, excess bleeding, and hematoma noted at insertion site.

Which type of pneumonia is best characterized by an infective agent that produces sputum samples with a peptidoglycan cell wall, expresses endotoxins, replicates readily in broth and on agar, grows in clusters, has pili, and does not stain when exposed to crystal violet?

Bacterial

Staphylococcus aureus commonly found in the skin, nares, and other body sites of clients without any signs and symptoms of infection is known as which condition?

Bacterial colonization

When talking about the various types of granulocytes, which granule contains heparin, an anticoagulant?

Basophils.

Following a collision while mountain biking, the diagnostic work up of a 22-year-old male has indicated the presence of an acute subdural hematoma. Which pathophysiologic process most likely underlies his diagnosis?

Blood has accumulated between the man's dura and subarachnoid space.

When explaining how carbon dioxide combines with water to form carbonic acid as part of an acid-base lecture, the faculty instructor emphasizes that which enzyme is needed as a catalyst for this reaction?

Carbonic anhydrase

The public health nurse is designing a course about risk factors for various chronic illnesses. For risk factors about which chronic illness will the nurse consult the Framingham study?

Cardiovascular disease

A middle-aged woman is brought to the emergency room after a minor auto accident. Her gait is staggering and unsteady, her speech is slurred, and she displays slight nystagmus. The police officer who brought her in says she has not been drinking. Her blood pressure is very high. Which health problem most likely underlies her present state?

Cerebellar damage caused by a cerebrovascular accident

The nurse will most likely assess which clinical manifestation in a client diagnosed with Creutzfeldt-Jakob disease?

Change in behavior and memory, loss of coordination leading to encephalopathy

A 16-year-old female has been brought to her primary care physician by her mother due to the girl's persistent sore throat and malaise. Which fact revealed in the girl's history and examination would lead the physician to rule out infectious mononucleosis?

Chest auscultation reveals crackles in her lower lung fields bilaterally.

A 4-year-old boy presents with a chronic cough and swollen lymph nodes. His records show that he has been given antibiotics several times in the past year with limited success, most recently for a liver abscess. He also has a recurring fungal skin condition. Which diagnosis is most likely the cause of this child's manifestations?

Chronic granulomatous disease

From the assigned team of clients, which client is at highest risk for the development of endothelial damage that may lead to disseminated intravascular coagulation (DIC)?

Client with chronic obstructive pulmonary disease (COPD) who is diagnosed with pneumonia, most likely from a virus

Which clent, when faced with acute stressful situations, would be considered highest risk for becoming noncompliant with his/her medication regimen?

Client with end-stage renal failure experiencing electrolyte imbalances related to having trouble sticking to the prescribed diet

A nurse practitioner is working in a crowded neighborhood where the population is primarily immigrants from China. The nurse has designed a research study to follow children from kindergarten to the age of 25. She is going to be looking at their diet, successful progression in school, health practices, and development of disease, to name a few items. This type of research is known as:

Cohort study

A nurse on a neurology unit is assessing a female brain-injured client. The client is unresponsive to speech, and her pupils are dilated and do not react to light. She is breathing regularly but her respiratory rate is 45 breaths per minute. In response to a noxious stimulus, her arms and legs extend rigidly. What is her level of impairment?

Coma

Several months ago, a 20 year-old male suffered a spinal cord injury brought about by a snowboard trick gone wrong. The lasting effects of his injury include flaccid bowel and bladder and the inability to obtain an erection. While sensation has been completely preserved in his legs and feet, his motor function is significantly impaired. What type of incomplete spinal cord injury has the man most likely experienced?

Conus medullaris syndrome

A 79-year-old female resident of an assisted living facility receives care from a community nurse on a regular basis for treatment of a chronic venous leg ulcer. Which factor would the nurse be most justified in ruling out as a contributing factor to the client's impaired wound healing?

Decreased antibody levels

Your client with end-stage renal disease is receiving 2 units of packed red blood cells for anemia (Hgb of 8.2 g/dL [82 g/L)). Twenty minutes into the first transfusion, the nurse observes the client has a flushed face, hives over upper body trunk, and is reporting pain in lower back. His vital signs include pulse rate of 110 and BP drop to 95/56. What is the nurse's priority action?

Discontinue the transfusion and begin an infusion of normal saline.

When a Rh-negative mother becomes pregnant by a partner who is Rh-positive, which advanced treatment modality will decrease the risk of the fetus developing hemolytic disease?

Discuss an intrauterine transfusion to the affected fetus.

The nurse is caring for a client with jaundice. What evidence will the nurse use to determine if the cause of the jaundice is due to a hemolytic condition?

Elevated unconjugated bilirubin

The nurse is caring for a client with a diabetic foot ulcer who has difficulty adhering to dietary restrictions. This has resulted in elevated blood glucose levels. What information is most important for the nurse to share with the client?

Elevations in blood glucose make it more difficult for your white blood cells to fight the infection.

A teenager, exposed to West Nile virus a few weeks ago while camping with friends, is admitted with headache, fever, and nuchal rigidity. The teenager is also displaying some lethargy and disorientation. The nurse knows which medical diagnosis listed below may be associated with these clinical manifestations?

Encephalitis

A clinician is providing care for a client who is immunocompromised following chemotherapy. The clinician knows that which characterization of the adaptive immune system is responsible for the client's disruption in normal immune function?

Epitopes on antigens are recognized by immunoglobulin receptors following presentation by accessory cells.

A 62-year-old female with a diagnosis of acute and chronic renal failure secondary to diabetes mellitus is receiving her weekly injection of epoetin, a supplementary form of erythropoietin. Which statement best captures the necessity of this medication?

Erythropoietin causes the erythrocyte colony-forming units to proliferate and mature.

Which phenomena best accounts for the increased presence of leukocytes at the site of inflammation?

Existing leukocytes stick to the epithelial cells and move along blood vessel walls.

A client with malignant melanoma has been prescribed alpha interferon, a biologic response modifier. Since this drug prolongs the cell cycle, increasing the percentage of cells in the G0 phase, and stimulates NK cells and T-lymphocyte killer cells, the nurse can anticipate that they may experience which common side effects?

Fever, chills, and fatigue

During science class, a student asks, "What is the difference between plasma and serum in the blood?" The nurse responds that the primary difference between plasma and serum is that plasma contains:

Fibrinogen

A blood sample is ordered to be collected in a plasma-separator tube, but the nurse can only find serum-separator tubes available. The nurse should be most concerned about the accuracy of which results if the serum-separator tube is used?

Fibrinogen and clotting factor levels

A client at risk for febrile neutropenia related to chemotherapy treatments has been prescribed medication to prevent this reaction. Which medication should the nurse be prepared to administer?

Filgrastim, a granulocyte-colony stimulating factor (G-CSF)

The blood work of a 44-year-old male client with a diagnosis of liver disease secondary to alcohol abuse indicates low levels of albumin. Which phenomenon would a clinician be most justified in anticipating?

Fluid imbalances

A patient is brought to the emergency department with reports of shortness of breath. Assessment reveals a full, bounding pulse, severe edema, and audible crackles in lower lung fields bilaterally. The clinician notifies the physician to obtain orders for which of these problems?

Fluid volume excess

The clinician is caring for a client with left-sided heart failure causing chronic activation of the renin-angiotensin-aldosterone system (RAAS). What is the clinician's priority assessment?

Fluid volume excess

A nurse researcher is interested in the natural history of a disease being studied. What should the nurse focus on to best understand the condition's natural history?

Focusing on clients who did not receive treatment for the condition

A 14-year-old boy, appearing to be intoxicated, arrives in the emergency department where the EMTs report the boy denies consuming anything out of the ordinary. However, an open antifreeze container was found in the boy's room. Which intervention is likely to be prescribed to treat the client's symptoms?

Fomepizole

A 63-year-old woman has begun a diet that purports to minimize the quantity and effects of free radicals in her body. Which physiologic process could best underlie her care provider's teaching about her new diet?

Free radicals increase cytokine expression and adhesion molecule levels, resulting in increased inflammation.

During a late night study session, a pathophysiology student reaches out to turn the page of her textbook. Which component of her nervous system contains the highest level of control of her arm and hand action?

Frontal lobe

The paramedic volunteering in the medical tent for a road race on a hot, humid day is asked to see a runner who has collapsed on the road. The paramedic notes he has sunken eyes, a temperature of 100°F (37.7°C), and dizziness. These are signs of a fluid volume deficit. Recognizing fluid volume deficit, which of these interventions does the paramedic carry out first?

Give him an electrolyte solution by mouth.

After several months on a waiting list, a 44-year-old male received a liver transplant 5 days ago. In the last 36 hours, he has developed a rash beginning on his palms and soles, along with abdominal pain and nausea. It has been determined by his care team that the immune response that is causing his symptoms originates not with his own compromised immune components but with those introduced with his new organ. This man's most likely medical diagnosis is:

Graft-versus-host-disease (GVHD)

Which glycoprotein is responsible for treating such diseases as bone marrow failure following chemotherapy and hematopoietic neoplasms such as leukemia?

Growth factors and cytokines.

During a period of stress, the clinician asks the client to close his eyes and think of a calm, relaxing place where he can feel the wind blowing on his cheek and smell the salty air from the ocean. This is an example of utilizing which type treatment for stress reduction?

Guided imagery

A 19 year-old intravenous drug user was exposed to the HIV virus 3 weeks ago and is experiencing a rapid proliferation in viral load. Which statement best captures an aspect of the process of HIV replication that underlies this proliferation?

HIV is able to change its RNA into DNA to allow for replication by CD4+ cells.

While teaching about HIV/AIDS to a group of high school seniors, the school health nurse will begin by explaining the basic facts. Which information will this likely include?

HIV is different from other viruses since it is a retrovirus that selectively attacks the body's immune cells.

A nurse is interviewing a client with fever, myalgia, headache, and lethargy. Which question is most important for the nurse to ask related to identifying the cause of these symptoms and the possible need for quarantining the client?

Have you or any family members traveled outside the country recently?

A 48-year-old man who has been HIV positive for 6 years has just learned that he has been diagnosed with Kaposi sarcoma (KS). Which fact most accurately conveys an aspect of his diagnosis?

He is likely to have lesions on his skin, mouth, or GI tract.

A 7-year-old boy is admitted to the hospital with a suspected diagnosis of lead toxicity. Which assessment finding is most congruent with the client's diagnosis?

Hemoglobin 9.9 g/dL (99 g/L)

A 39-year-old female with HIV has been characterized as a typical progressor by her care team, and is experiencing an increase in her manifestations and complaints as her CD4+ count declines. Which health problem would her care team most likely attribute to a cause other than her HIV?

Her decreased bone density and recent fractures

Which assessment and laboratory findings would be most closely associated with acute leukemia?

High blast cell counts and fever

A particular disease has a debilitating effect on the ability of sufferers to perform their activities of daily living, and is a significant cause of decreased quality of life. However, few people die as a result of the disease's direct effects. There are hundreds of thousands of Americans living with the disease but relatively few new cases in recent years. Which statement best conveys an accurate epidemiologic characterization of the disease?

High morbidity; low mortality; high prevalence, low incidence.

A college student has been experiencing frequent headaches that he describes as throbbing; he is reporting difficulty concentrating while studying. Upon cerebral angiography, he is found to have an arteriovenous malformation. Which pathophysiologic concept is likely responsible for his symptoms?

High pressure and local hemorrhage of the venous system

A 60-year-old male office worker presents to a clinic complaining of new-onset of lower back pain that has been worsening over the last 6 weeks. The physician knows that which component of his physical assessment and history is most indicative of a serious pathologic process (like aortic aneurysm or cancer)?

His onset of pain has been gradual and he has no prior history of lower back problems.

A deficiency in which of these would result in an inhibition of the inflammatory response?

Histamine

When explaining to parents what is occurring when their child has an acute bronchial asthma attack, the respiratory therapist will emphasize that which mediator is primarily responsible for the bronchial constriction?

Histamine

When explaining the final stages of the inflammatory response to pathogens, the nurse should include which educational topics?

How the body can kill the pathogen by generating toxic oxygen and nitrogen products, producing such things as nitric oxide and hydrogen peroxide

While undergoing a kidney transplant from a non-family member, the client's transplanted kidney has just had the arterial clamps removed. The OR staff notices that the organ is turning purple with no urine output. When explaining to the family why they had to remove the donor kidney, the surgeon would likely include which statement?

Hyperacute rejection occurs because antibodies against HLA antigens are deposited in vessels, causing necrosis.

A clinician working in a busy orthopedic clinic is asked to perform the Tinel sign on a client having problems in her hand/wrist. In order to test Tinel sign, the clinician should give the client which direction?

I'm going to tap (percuss) over the median nerve in your wrist; tell me what sensation you feel while I am doing this. Does the sensation stay in the wrist or go anywhere else?

A client arrives in the emergency department reporting muscle cramps and spasms in the legs and feet; tingling in the fingers, toes, and lips; and anxiety. Serum calcium level is 4.8 mg/dL (1.2 mmol/L). Suspecting tetany, which medication should clinicians be prepared to administer?

IV calcium gluconate

While taking a daily walk, the nurse is asked by a neighbor what centric fusion (Robertsonian) translocation means. The neighbor tells the nurse that a family member has been diagnosed with this and is now afraid to have children. Given this diagnosis, what may be potential risks for her offspring?

If chromosome 21 is involved, there is a high risk for producing a child with Down syndrome.

The nurse is caring for an 8-month-old male client who has just been diagnosed with hemophilia A. The parents ask about the likelihood that any future children would be born with the condition. What is the nurse's best response?

If you have another son there is a 50% chance that he will also have hemophilia.

The parent of a child with allergies has been told the child will have a blood test done for immunoglobulin measurement. Which immunoglobulin will be tested?

IgE

Three days ago, a mother delivered her full-term infant who had been identified as having an in utero infection. The infant is receiving antibiotic and phototherapy, and the mother is breast-feeding. Which types of immunoglobulins could most reasonably be expected to predominate in the infant's immune system?

IgG, IgA, IgM

While being on subcutaneous heparin injections for deep vein thrombosis during the third trimester of her pregnancy, a client begins to experience major side effects. Her OB physician has called in a specialist who thinks the client is experiencing heparin-induced thrombocytopenia. The nurse should anticipate which order?

Immediately discontinue the heparin therapy.

A 51-year-old female has been found to have metastatic lesions in her lung but her oncologist is unsure of the primary tumor site. Which procedure is most likely to aid in this determination?

Immunohistochemistry

When educating a client with a wound that is not healing, the nurse should stress which dietary modifications to ward off some of the negative manifestations that can occur with inflammation?

Increase your intake of oily fish and fish oil so that you will increase absorption of omega-3 polyunsaturated fatty acids.

A client with poorly controlled diabetes mellitus presents to the emergency department with suspected ketoacidosis. Which diagnostic results would be most likely to confirm this diagnosis?

Increased anion gap, base deficit

A client who has been awaiting the results of a bone marrow biopsy for several days is experiencing stress as a result of uncertainty and the possibility that abnormal cell growth may be detected. A physical examination and blood work would most likely yield which result?

Increased blood pressure and heart rate; increased antidiuretic hormone (ADH).

Health care team members know recently an increased incidence of infections such as West Nile virus and SARS do not match previously established patterns. Which phenomenon constitutes the most significant contributor to the spread of new diseases in the United States?

Increased ease and speed of travel for individuals and groups

Which aspect of a client's site of inflammation would help the care provider rule out chronic inflammation?

Increased neutrophils

Following a course of measles, a 5-year-old girl developed scattered bruising over numerous body surfaces and was diagnosed with immune thrombocytopenic purpura (ITP). As part of her diagnostic workup, blood work was performed. Which result is most likely to be considered unexpected by the health care team?

Increased thrombopoietin levels

The geriatrician providing care for a 74-year-old man with diagnosis of Parkinson disease has recently changed the client's medication regimen. What is the most likely focus of the pharmacologic treatment of the man's health problem?

Increasing the functional ability of the underactive dopaminergic system

A nurse practitioner is providing care for a client with low levels of the plasma protein gamma globulin. The nurse would recognize that the client is at risk of developing which health problem?

Infections

A 40-year-old woman who experiences severe seasonal allergies has been referred by her family physician to an allergist for weekly allergy injections. The woman is confused as to why repeated exposure to substances that set off her allergies would ultimately benefit her. Which phenomenon best captures the rationale for allergy desensitization therapy?

Injections of allergens simulate production of IgG, combining with the antigens to prevent activation of IgE antibodies.

Which statement most accurately conveys an aspect of cell injury due to impaired calcium homeostasis?

Injured cells tend to accumulate calcium.

A 44-year-old hospitalized client with a diagnosis of end-stage acquired immunodeficiency syndrome (AIDS) has been placed on neutropenic precautions that limit his interaction with visitors, staff, and other clients. What is the underlying rationale for these precautions?

Insufficient levels of neutrophils make him particularly susceptible to infections.

A 61-year-old client with a 40 pack-year history of cigarette smoking and chronic obstructive pulmonary disease (COPD) is experiencing an increase in arterial levels of CO2. This change stimulates the inspiratory center in the medulla oblongata, which in turn causes the diaphragm to contract more forcefully and increase respiratory rate. Which term best describes the role of the inspiratory center?

Integrator

The nurse should anticipate that she will need to teach the client newly diagnosed with multiple sclerosis how to give injections if which medication to assist with reducing exacerbations is prescribed?

Interferon beta

The nurse is caring for a client who has been on an oral corticosteroid for several years for an autoimmune condition. The nurse notes several bruises of different stages of resolution on the client. What is the nurse's best response to this finding?

Interview the client regarding side effects of corticosteroid therapy.

A clinician is providing care for a client who has been diagnosed with metabolic alkalosis after several days of antacid use. Which treatment should the clinician be prepared to give?

Intravenous administration of a KCl solution

Misinterpreting her physician's instructions, a 69-year-old woman with a history of peripheral artery disease has been taking two 325 mg tablets of aspirin daily. How has this most likely affected her hemostatic status?

Irreversible acetylation of platelet cyclooxygenase activity has occurred.

A student is attempting to trace the feedback cycle involved in the stress response. Which neural structure is thought to be the central integrating site for the stress response?

Locus caeruleus

A person who has been diagnosed with HIV infection 12 years ago and still has a CD4+ cell count of 800 cells/μL and a low viral load is considered to be in which clinical group?

Long-term nonprogressor

A 16-year-old girl has broken her arm while snowboarding and is shocked at the amount of swelling at the injury site. Which statement best explains the physiologic rationale for her swelling?

Loss of plasma proteins causes an increase in interstitial osmotic pressure.

A client with a diagnosis of end-stage liver failure has arterial blood gas results indicating altered pH. Which of these consequences of liver failure has most likely caused the acid-base disturbance?

Low albumin and plasma globulin levels.

Which component is needed for long-lasting immunity in a client with a diagnosis of sepsis without the causative agent identified?

Lymphocytes

A 53-year-old female hospital client has received a kidney transplant following renal failure secondary to hypertension. The teaching prior to transplant made her aware that she would need to take anti-rejection drugs for the rest of her life. Which aspect of the immune system underlies this necessity?

MHC molecules will never develop in the cells of the donor organ and effector cells will be continually stimulated.

A client presents with sudden, violent diarrhea and vomiting after consuming chicken and potato salad 8 hours prior at the beach on a hot day. The nurse knows which statement best matches the phase of the infectious process of food poisoning?

Maximum impact of infectious process

Which diagnostic finding is likely to result in the most serious brain insult?

Mean arterial pressure (MAP) that equals intracranial pressure (ICP)

A client's arterial blood gases reveal normal oxygen level, pH 7.50, PCO2 level of 50 mmHg (6.65 kPa) and HCO3 level of 30 (30 mmol/L). The client's respiratory rate is 12 breaths/min and all other vital signs are within normal range. What is this client's most likely diagnosis?

Metabolic alkalosis

An adult female, newly diagnosed with possible leukemia, arrives for a bone marrow biopsy. The admission laboratory work includes: red blood cell count (RBC): 3.4 ×106/μL (3.4 ×1012/L); white blood cell count (WBC): 2,500/μL (2.5 ×109/L); platelet count of 80 ×103/μL (80 ×109/L); all below normal values. What assessment is the highest priority for this client?

Monitoring for any type of bleeding.

A 30 year-old male's blood work and biopsies indicate that he has proliferating osteoclasts that are producing large amounts of IgG. What is the man's most likely diagnosis?

Multiple myeloma

The nurse in the emergency department knows clients exposed to Clostridium botulinum, an agent of bioterrorism, would likely be exhibiting which clinical manifestation listed below?

Muscle weakness in extremities eventually leading to paralysis of respiratory muscles

A client with a diagnosis of schizophrenia has been admitted with suspected hyponatremia after consuming copious quantities of tap water. Which finding does the nurse anticipate uncovering related to this problem?

Muscle weakness, lethargy and headaches.

The physician is caring for a client with a longstanding diagnosis of hypocalcemia secondary to kidney disease. The physician observes which clinical manifestations in this client?

Muscular spasms and reports of tingling in hands/feet

An 81-year-old male client who has a diagnosis of orthostatic hypotension is experiencing an episode of particularly low blood pressure. The man's body has responded by increasing levels of angiotensin II in the bloodstream, a hormone which decreases glomerular filtration rate in the kidneys and contributes to an increase in blood pressure. Which phenomena best describes what has occurred?

Negative feedback

A client presented to the emergency department of the hospital with a swollen, reddened, painful leg wound and has been diagnosed with methicillin-resistant Staphylococcus aureus (MRSA) cellulitis. The client's physician has ordered a complete blood count and white cell differential. Which blood component would the physician most likely anticipate to be elevated?

Neutrophils

Following an injury resulting in a small cut from a knife, the first cells to go to the area of the cut would be the:

Neutrophils

A student arrives at the health clinic anxious and afraid. The student found an enlarged lymph node in the groin area that is extremely painful to touch and "knows" it is cancer. What information should the health care provider relay to this student about lymphadenitis?

Not all swollen lymph nodes are due to cancer. It could be caused by an infection in the genital region.

Clinicians at a long-term care facility encourages the older adults to drink even though they may not feel thirsty at the time. Which statement supports this action?

Older adults often experience a decrease in the sensation of thirst, even when serum sodium levels are high.

The clinician is caring for a client who is suffering from high levels of chronic stress. On what should the clinician focus, as a negative effect on the client's health due to chronically elevated levels of cortisol?

Osteoblast activity and protein synthesis are suppressed to refocus energy.

After being thrown off the back of a bull, a bull rider can move his arms but has loss of motor function in the lumbar and sacral segments of the spinal cord. This is referred to as:

Paraplegia

An 80-year-old woman is slated for total hip replacement the following day, and is experiencing a large amount of stress around her potential surgical outcomes. Which is most likely to be uninvolved in the physiologic response to her stress?

Parathyroid

A client with hypercalcemia and hypercalciuria has just passed a kidney stone. The clinician recognizes which of these laboratory studies should also be assessed?

Parathyroid hormone level

A client who has a diagnosis of an autoimmune disease asks his physician why it is that their immune system does not attack all of the cells that make up his body. Which of the following aspects of pathogen recognition in the innate immune system listed below would underlie the physician's response?

Pattern recognition receptors (PRRs) ensure cells are correctly identified.

When discussing the sequence of clot dissolution, the science instructor will talk about which item that begins the process?

Plasminogen

A 71-year-old male client with a history of myocardial infarction (MI) and peripheral vascular disease (PVD) has been advised by his family physician to begin taking 81 mg aspirin once daily. Which statement best captures an aspect of the underlying rationale for the physician's suggestion?

Platelet aggregation can be precluded through inhibition of prostaglandin production by aspirin.

As part of a community class, student nurses are developing curriculum to teach expectant parents the importance of having their child properly secured in a child safety seat. During the class, the students are going to have a safety officer examine the car seats that the parents have installed in their vehicle. This is an example of which type of prevention?

Primary prevention

Which statements best captures an aspect of the process of hematopoiesis?

Progenitor cells differentiate into precursor cells.

A 29-year-old construction worker got a sliver under his fingernail 4 days ago. The affected finger is now reddened, painful, swollen, and warm to the touch. Which of the following hematologic processes is most likely occurring in the bone marrow in response to the infection?

Proliferation of immature neutrophils

A client is asked to stand with feet together, eyes open, and hands by the sides. Then the client is asked to close the eyes while the clinician observes for a full minute. What assessment is the clinician performing?

Proprioception

A client has been suffering from fatigue and the health care provider suspects anemia. Which test results will the health care provider consult when investigating the cause of the anemia?

RBC indices

A client with a history of epilepsy is brought to the emergency department due to seizure activity. What information about the client is most important for the nurse to collect?

Recent medication history

A client has been inhaling viruses periodically while on a cross-country flight. Which situation listed below would most likely result in the stimulation of the client's T lymphocytes and adaptive immune system?

Recognition of a foreign peptide bound to a self-major histocompatibility complex (MHC) molecule

A client has been diagnosed with a brain tumor that cannot be removed surgically. During each office visit the clinician will be assessing the client for syndrome of inappropriate antidiuretic hormone (SIADH). Which assessment would alert the clinician that the client may be developing this complication?

Report of decreased urine output, no edema noted in ankles, increasing headache

In the neurotrauma unit, a teenager with a closed head injury related to an automobile accident is experiencing high intracranial pressure (ICP). He is intubated and on a ventilator. One treatment for this is to allow him to progress into which acid-base imbalance in an attempt to lower ICP?

Respiratory alkalosis.

If an Rh-negative mother is giving birth to an Rh-positive infant, the nurse should be prepared to administer:

Rh immune globulin

Which breakfast diet contains chemicals that likely can contribute to a higher rate of cancer risk?

Sausage fried in reused grease and biscuits and gravy.

A CT Technologist has developed signs and symptoms of a latex sensitivity, and is undergoing allergy testing as well as blood work. Which component of the Tech's blood work would most likely be the focus of her health care provider's analysis?

Serum IgE immunoassays

The nurse caring for a lung cancer client with metastasis to the brain suspects the client has developed a paraneoplastic syndrome known as Syndrome of Inappropriate Antidiuretic Hormone (SIADH) secretion. Which laboratory result in this client who has gained 3 lb in a day would alert the nurse to the possibility of SIADH?

Serum sodium of 115 mEq/L (115 mmol/L)

A 77-year-old female hospital client has contracted Clostridium difficile during her stay and is experiencing severe diarrhea. Which statement best conveys a risk that this woman faces?

She is susceptible to isotonic fluid volume deficit.

A 66-year-old female client has presented to the emergency department because of several months of intermittently bloody stool which has recently become worse. The woman has since been diagnosed with a gastrointestinal bleed secondary to overuse of nonsteroidal anti-inflammatory drugs that she takes for her arthritis. The health care team would realize that which situation is most likely?

She will have iron-deficiency anemia due to depletion of iron stores.

The nurse is performing discharge teaching for a client who has recovered from sepsis following an open cholecystectomy. What should be the nurse's primary focus?

Signs and symptoms of infection

Which statement is an accurate descriptor of the role of viruses in human infections?

Some viruses are capable of transforming normal host cells into malignant cells.

A client has been identified as having an excess of macrophage inhibitory factor, causing the client to have inhibited movement and activity of macrophages. Which process listed below would the health care team member expect to remain unaffected?

Specificity and memory of the immune response

A child has been diagnosed with thalassemia. Which of the following comorbidities may occur as a result of having thalassemia?

Splenomegaly

A multidisciplinary healthcare team operates a program aimed at the prevention, identification, and treatment of diabetes on a large Indian reservation. Which aspect of the program would most likely be classified as secondary prevention?

Staffing a booth where community residents who are attending a baseball tournament can have their blood glucose levels checked.

A hospital client is receiving intravenous infusion of heparin for treatment of a pulmonary embolus. Which phenomenon is most likely occurring, resulting in the drug's therapeutic effect?

Suppression of fibrin formation

The cells primarily programmed to remove the invading organisms and remember the antigen to respond rapidly during the next exposure are:

T-lymphocytes and B-lymphocytes.

Which enzyme listed below is responsible for the ability of cancer cells to resist aging and contributes to cellular immortality that is so characteristic of this disease process?

Telomerase

An occupational therapist conducts a group therapy program called MindWorks with older adults who have diagnoses of dementia and Alzheimer's disease. The goal of the group is to slow the cognitive decline of clients by engaging them in regular, organized mental activity such as reading maps and solving puzzles. How would the program most likely be characterized?

Tertiary prevention

A 6-year-old girl with a diagnosis of Marfan syndrome is being assessed at a community health clinic. Which assessment would be the health care professional's lowest priority?

Tests of kidney function

An occupational health nurse working with police officers wishes to apply Selye's general adaptation syndrome theory in practice. Which concept should the nurse apply?

The alarm stage involves the release of cortisol and catecholamines.

A hospital laboratory technician performing routine blood analysis as part of an inpatient's assessment is examining the sample in a test tube following processing in a centrifuge and the addition of an anticoagulant. Which observation would the technician most likely interpret as an anomaly?

The bottom layer of blood in the tube accounts for around 40% to 45% of the total volume.

A 6-year-old boy with intellectual disability secondary to fragile X syndrome has been admitted to the hospital with a mitral valve prolapse. The health care worker caring for the child should have which concepts as part of her knowledge base regarding fragile X syndrome?

The boy will pass the gene to all his future daughters, who will become carriers.

A client asks the health care provider why his lower legs look purple. The health care provider will base the response on which pathophysiologic principle?

The bruising around the ankles is due to the fact that it is a dependent area where the capillary pressure is higher.

A 2-year-old girl has had repeated ear and upper respiratory infections since she was born. A pediatrician has determined a diagnosis of transient hypogammaglobulinemia of infancy. What is the physiologic origin of the child's recurrent infections?

The child's immune system is unable to synthesize adequate immunoglobulin on its own.

A clinician is conducting an assessment of a male client suspected of having a disorder of motor function. Which assessment finding would suggest a possible upper motor neuron (UMN) lesion?

The client displays increased muscle tone.

A counseling psychologist is working with a 30-year-old female client who is experiencing the symptoms of posttraumatic stress disorder (PTSD) following a house fire several months prior. Which of the client's diagnostic results could most likely be interpreted as a manifestation of PTSD?

The client has decreased levels of cortisol.

When explaining to a client admitted for stress-induced supraventricular tachycardia, the clinician will incorporate which statement about what happens in the body as a result of excess stress?

The corticotrophin-releasing factor stimulates the release of norepinephrine, which is responsible for "fight-or-flight" reaction to stress.

An electrician who has been working 14- to 16-hour days for several weeks to ensure the financial survival of his business presents to his family physician with a report of persistent headaches and insomnia. His family physician attributes the physical symptoms to the ongoing stress likely caused by which component of the stress response?

The fact that he has previously had difficulty coping with stress

The clinical educator of a hospital medical unit has the mandate of establishing evidence-based practice guidelines for the nursing care on the unit. Which statement most accurately captures a guiding principle of the nurse's task?

The guidelines will combine individual expertise with external systematic evidence.

A client is suspected of having an infection, but the microorganism suspected as the cause cannot be cultured. The client asks how the health care provider will identify the organism. What is the nurse's best response as to the methods used?

The health care provider will introduce cultured, marked antibodies to your blood sample and observe for a reaction with antigens.

A 77-year-old client diagnosed with chronic obstructive pulmonary disease (COPD) is experiencing impaired gas exchange and CO2 retention, despite a rapid respiratory rate. Which pathophysiologic principle would the health care team expect if the client's compensatory mechanisms are working?

The kidneys will adapt with an increase in plasma HCO3- and the pH will increase.

A 60-year-old woman is suspected of having non-Hodgkin lymphoma (NHL). Which aspect of her condition would help to rule out Hodgkin lymphoma?

The lymph nodes involved are located in a large number of locations in the lymphatic system.

Which physiologic process would be considered a positive feedback mechanism?

The platelet-aggregation mechanism activating clot formation

A 77-year-old male client with a diagnosis of stomach cancer has been found to have metastases in his liver. The client and his family are surprised at this turn of events, stating that they don't see how he could have developed cancer in his liver. Which fact would underlie the reply that the care team provides?

The portal circulatory system brings venous blood from the GI tract into the liver.

Following a biopsy, a 54-year-old man has been diagnosed as having a benign neoplastic tumor. Which characteristic most likely applies to his tumor?

The well-differentiated, neoplastic cells are clustered together in a single mass.

Which statement best conveys a characteristic of red blood cells?

They contribute to the maintenance of blood pH.

As part of an orientation to a genetic counseling practice, a group of medical students are differentiating between autosomal recessive disorders and autosomal dominant disorders. Which statement is true of autosomal recessive disorders?

They tend to have a more uniform symptomology than autosomal dominant disorders.

The nurse is caring for a 30-year-old woman who is diagnosed with iron deficiency anemia. The client is confused how this condition developed because she ate many foods high in iron. How should the nurse respond?

This iron deficiency anemia was most likely the result of chronic blood loss over a period of time.

A 53-year-old man presents with inability to concentrate, itching in his fingers and toes, elevated blood pressure, and unexplained weight loss. He is diagnosed with primary polycythemia. What will be the primary goal of his treatment?

To reduce the viscosity of his blood

New parents are upset their 2-day-old infant is requiring phototherapy for hyperbilirubinemia. Which factors would the pediatrician be most likely to rule out as a contributor to high bilirubin levels?

Transitioning of hemoglobin F (HbF) to hemoglobin A (HbA)

During a myocardial infarction (MI), a client with a 97% occlusion of the left descending artery develops ventricular dysrhythmias due to the amount of ischemia occurring in the myocardium. While providing education about MI's, which statement is most accurate to share with this client?

Treatment needs to be sought immediately so that the buildup of lactic acid is limited and cellular changes can be reversed.

A 1-year-old child who has experienced low platelet counts and bacterial susceptibility has been admitted to a pediatric medical unit of a hospital for treatment of Wiskott-Aldrich syndrome. The clinician who has admitted the child to the unit would anticipate which short-term and longer-term treatment plans?

Treatment of eczema; management of bleeding; bone marrow transplant

A 70 year-old male has been diagnosed with a stroke that resulted in an infarct to his cerebellum. Which clinical finding would be most closely associated with cerebellar insult?

Unsteady gait and difficulty speaking and swallowing

Which process would most likely be considered an anomaly during the cellular phase of inflammation?

Vasoconstriction

A client in the intensive care unit who has a brain tumor has experienced a sharp decline. The care team suspects that water and protein have crossed the blood-brain barrier and been transferred from the vascular space into the client's interstitial space. Which diagnosis best captures this pathophysiology?

Vasogenic edema

A 14-year-old boy has been diagnosed with infectious mononucleosis. Which pathophysiologic phenomenon is most responsible for his symptoms?

Viruses are killing some of his B cells and becoming incorporated into the genome of others.

A 44-year-old female client presents to the emergency department with abnormal bleeding and abdominal pain that is later attributed to gallbladder disease. Which diagnosis would the medical team be most justified in suspecting as a cause of the client's bleeding?

Vitamin K deficiency

Which procedure reduces the potential for infection primarily by addressing the portal of entry?

Wearing gloves when contact with blood or body fluids is anticipated

A client with a large decubitus ulcer asks the nurse, "How it is possible for a wound this deep to ever fully heal?" What is the nurse's best response?

With adequate resources, the body is capable of regenerating tissue and blood flow to the area over time.

A family of five vacationed together, but only the client contracted traveler's diarrhea. The client asks the nurse, "We all ate and drank the same things. Why am I the only one with this condition?" What should the nurse offer to the client as the most likely reason for susceptibility for this condition?

You are taking a proton pump inhibitor for gastric reflux disease.

A client has been diagnosed with rheumatoid arthritis and asks what causes this condition. What is the best response?

Your immune system is attacking your own tissues as if they were harmful organisms.

The clinician is caring for client who has been living with Parkinson disease for the past 10 years and is being treated with levodopa-carbidopa. The client often leaves the unit with family for extended periods. What should be prioritized when teaching the client?

Your medication needs to be taken at equal intervals to reduce symptom fluctuation.

When counseling a male client with suspected HIV, the physician informs him that if the enzyme-linked immunosorbent assay (ELISA) comes back positive, then:

a second test known as the Western blot assay will be ordered to confirm positive HIV status.

A Cardiac Rehab specialist is developing a program to help clients reduce the impact of chronic stress on their health. Which clients should the clinician prioritize as being able to most benefit from the program? Select all that apply.

a. A 30-year-old with substance use b. A 22-year-old with an eating disorder c. A 45-year-old with inflammatory bowel disease

The clinician is caring for a client who suffered a spinal injury at T4 several years ago. The client develops a flushed neck, reports feeling unwell, and has an elevated blood pressure. What are the clinician's priority actions? Select all that apply.

a. Assess recent urine output. b. Remove compression stockings. c. Sit client in an upright position.

The clinician is caring for a client who has a tumor that results in excess levels of catecholamines being released. Which assessments will be prioritized in this client's plan of care? Select all that apply.

a. Blood pressure b. Heart rate and rhythm c. Blood glucose

After years of going to different physicians with vague symptoms, a 55-year-old client with a history of Hodgkin lymphoma has been diagnosed with a secondary immunodeficiency syndrome. The client asks what this means. Secondary immunodeficiency disorders may occur if which background information is available? Select all that apply.

a. Can occur in clients taking corticosteroids daily. b. May be a result of chemotherapy being used to treat a cancer c. Usually develop later in life

The nurse is assessing a client. What findings should the nurse apply as evidence of iron deficiency anemia? Select all that apply.

a. Concave fingernails (koilonychia) b. Dyspnea on exertion c. Oral lesions

Tumor necrosis factor-α and IL-1 are major cytokines that mediate inflammation. If the client is developing a systemic response to an infection, the nurse will likely assess which clinical manifestations? Select all that apply.

a. Elevated temperature b. Tachycardia c. Anorexia

The nurse notes eosinophilia on the client's laboratory results. The nurse should interview the client about a history of which conditions? Select all that apply.

a. Exposure to parasites b. Allergic reactions c. Asthma

A brain tumor causing clinical manifestations of headache, nausea, projectile vomiting, and mental changes is likely located in which part of the brain? Select all that apply.

a. Frontal lobe. b. Intra-axial c. Extra-axial

A client arrives at the emergency department with symptoms of stroke. What evidence should the nurse gather to determine if the client is a candidate for thrombolytic therapy? Select all that apply.

a. History of stroke b. Blood pressure c. Time of symptom onset

A school nurse is working with a 16-year-old client recovering from mononucleosis. Which activities should the nurse recommend the teenager avoid while recovering? Select all that apply.

a. Hockey b. Football

A client has started on interferon beta for treatment of multiple sclerosis. What should the clinician include in the teaching? Select all that apply.

a. Interferon beta alters your immune response. b. The medication will be given by subcutaneous injection.

A client arrives for a scheduled bone marrow aspiration and biopsy. The nurse explains to the client that which position(s) is best for accessing the posterior iliac crest for aspiration? Select all that apply.

a. Lateral decubitus b. Prone

The Respiratory Therapist is concerned with her client's arterial blood gas (ABG) results-especially the pH 7.30; and PCO2 49 mmHg (6.52 kPa). The respiratory therapist interprets these ABG results and assesses her client for which clinical manifestations of respiratory acidosis? Select all that apply.

a. Muscle twitching b. Headache with complaints of blurred vision

The nurse is caring for a client with leukemia and a blast count of 100,000 cells/μL (100 x 109/L). What client reports should prompt the nurse to take immediate action? Select all that apply.

a. New onset dyspnea b. Headache c. Lethargy

Which assessment finding(s) correlatea with the health care providers preliminary diagnosis of a more aggressive form of non-Hodgkin lymphoma (NHL)? Select all that apply.

a. Night sweats b. Elevated temperature c. Weight loss

A client visited his health care provider after finding an enlarged lymph node along the mediastinal border. The health care provider may be alerted to a possible diagnosis of Hodgkin lymphoma (HL) based on the client having which other clinical manifestations? Select all that apply.

a. Night sweats b. Unexplained pruritus

An 80-year-old client has a stage 3 decubitus ulcer on the left ischial tuberosity which has not shown much improvement despite optimal local wound treatment. What other interventions should the nurse recommend to promote wound healing? Select all that apply.

a. Nutritional supplements b. A review of the client's medications c. Placing the client on an alternating pressure mattress

A 41-year-old woman diagnosed with multiple sclerosis (MS) is sharing her story with members of an MS support group made up of people recently diagnosed. Which aspects of her health problem should the woman warn others to expect at some point in the progression of the disease? Select all that apply.

a. Progressive loss of visual acuity b. Debilitating fatigue c. Loss of mental acuity

The nurse is planning care for a client in the acute recovery phase after an ischemic stroke. What interventions will the nurse prioritize when planning care? Select all that apply.

a. Swallowing precautions b. Falls prevention c. Deep vein prophylaxis

The nurse is performing passive range-of-motion exercises with a client who suffered an ischemic stroke 2 weeks ago and has hemiparesis. The client says, "What is the point of doing these exercises if I will never be able to use that arm again?" What should the nurse include when responding to the client? Select all that apply.

a. These exercises help to prevent painful shoulder complications after a stroke. b. You may still regain use of this arm, and these exercises may help with this recovery. c. We want to ensure your shoulder joint remains flexible while you recover.

A client with leukemia begins hemorrhaging from every orifice. The physician is concerned that the client has developed disseminated intravascular coagulation (DIC). The nurse should anticipate which order to be prescribed for this client? Select all that apply.

a. Transfuse fresh frozen plasma. b. Transfuse 2 units of platelets.

A newly diagnosed HIV-positive adolescent has blood work drawn that includes a CD8+ T-cell count. Which functions of CD8+ T cells listed below will assist the adolescent's immune system in fighting off the viral attack? Select all that apply.

a. Trigger intracellular programmed death b. Release of destructive enzymes

A female client arrives at the clinic reporting fatigue that is exhausting, bruising on the skin, and bleeding from the gums. Which diagnostic result(s) or history finding(s) correlates with a diagnosis of aplastic anemia? Select all that apply.

a. White blood cell (WBC) count <1,000/μL (1 ×109/L) b. Recent bout with mononucleosis viral infection c. Platelet count 50 ×103/μL (50 ×109/L) d. Completed chemotherapy for leukemia in the past year

Which clients are at increased risk for developing secondary hypogammaglobulinemia, a secondary humoral immunodeficiency? Select all that apply.

a. malnourished child who only drinks cow's milk b. client witha a history of seizures controlled by an antiepileptic c. cleint with burns covering more than 45% of the body

The nurse is caring for a client with traumatic brain injury and increased intracranial pressure. What findings should the nurse report due to the risk for increasing intracranial pressure? Select all that apply.

a. pH of 7.25 b. PaCO2 of 50 mm Hg (6.65 kPa) c. PaO2 of 70 mm Hg (9.31 kPa)

A clinical nurse specialist is interested in developing a research study focused on clients living with the sequelae of ischemic stroke. Which clients should the nurse include? Select all that apply. The nurse should include clients with:

a. vision changes b. dysphagia (difficulty swallowing) c. unilateral weakness.

High incidences of infectious illnesses among the older adults who reside in a long-term care facility are most likely to have diminished immune capacity because of:

decreased numbers and responsiveness of T lymphocytes.

A client diagnosed with low-risk chronic lymphocytic leukemia (CLL) has recently developed thrombocytopenia. One of the medications utilized to treat this would be:

dexamethasone, a corticosteroid.

The spirochete Leptospira is primarily transmitted to farmers by:

direct contact with infected animals.

A client has experienced a pontine stroke which has resulted in severe hemiparesis. What priority assessment should the nurse perform prior to allowing the client to eat or drink from the food tray?

gag reflex assessing for problems with chewing and swallowing

A client with Parkinson disease has challenged himself to maintain mobility for longer than the physician predicts. He strives every day to walk 5 to 10 steps farther than the day before. This phenomenon, being researched by social psychologists, is known as:

hardiness

Which client's signs and symptoms would allow a clinician to be most justified in ruling out stroke as a cause? An adult:

has had a gradual onset of weakness, headache, and visual disturbances over the last 2 days.

The nurse knows which statement best describes a characteristic trait of Rickettsiae related to Rocky Mountain spotted fever? Rickettsiae:

have both RNA and DNA.

A 10 year old child with strep throat asks the nurse, "Why are there large bumps [lymph nodes] on my neck when my throat gets sore?" The clinician replies that lymph nodes:

help your body fight off infections by allowing special cells [lymphocytes and macrophages] to move through the lymph chain and engulf and destroy germs.

A school-age child with a history of asthma has brought a note home from school stating there has been one case of meningitis (Neisseria meningitidis) in the school. Since the mother is a nurse, she is very concerned since she knows the portal of entry of this pathogen is:

inhalation via the respiratory tract such as through breathing or yawning.

The nurse is collecting a blood sample for an ordered CBC. The required tube contains EDTA so the nurse will:

invert the tube several times to prevent the blood from coagulating.

A pregnant client is having premature labor cramps and is diagnosed with preeclampia. The admitting prescriptions include starting an IV infusion of magnesium sulfate. What intervention will the clinicians provide this client?

monitoring for changes in neuromuscular status like lethargy, confusion, or hyporeflexia

The school nurse has several children with hemophilia A. After recess, one hemophilia student comes to the school nurse complaining of pain in the knee from falling on the playground. The nurse notes there is swelling in the knee and pain on palpation. The nurse should:

notify parents to pick up the child and possibly administer Factor VIII.

Which statement listed below relative to a client with malignant melanoma treated with alpha interferon (IFN-α) is accurate? Alpha interferon (IFN-α):

plays an important role in the modulation of the inflammatory response.

Which client is most likely to have impairments to the wound healing process? A client with:

poorly controlled blood sugars with small blood vessel disease.

A 5-year-old needs to undergo a bone marrow biopsy. When educating the parents, which site should the nurse state is the most commonly used?

posterior iliac crest

When looking at a granulocyte under a microscope, the anatomy student would describe it as:

shaped like a sphere with multilobar nuclei.

A health care provider has prescribed blood products for a trauma client with a history of selective immunoglobulin A deficiency (SIGAD) who is going into hypovolemic shock. Which blood product is most appropriate to infuse?

specially washed erythrocytes from normal donor

A geriatrician is following a number of clients on a subacute geriatric medical unit, some of whom require diagnostic blood work. Which client would be most likely to have an erythrocyte sedimentation rate (ESR) screening test ordered? An adult with:

systemic lupus erythematosus.

A drug in a category identified as a colony-stimulating factor (CSF) helps:

to stimulate bone marrow to produce large numbers of mature cells such as platelets and erythrocytes.

Which individual is likely to have the best prognosis for recovery from his or her insult to the peripheral nervous system? An adult:

who had his forearm partially crushed by gears during an industrial accident.

Two nursing students are attempting to differentiate between the presentations of immune thrombocytopenic purpura (ITP) and thrombotic thrombocytopenic purpura (TTP). Which of the students' statements best captures an aspect of the two health problems?

"Both of them involve low platelet counts, but in TTP there can be more, not less, hemostasis.

A community health nurse is conducting an education session with a group of pregnant women. One of the clients states, "I am not feeling right about getting a flu vaccine while pregnant." What is the nurse's best response?

"By getting vaccinated, you reduce the risk of you and your baby getting influenza."

The family of an older adult client is wondering why his "blood counts" are not rising after his last GI bleed. They state, "He has always bounced back after one of these episodes, but this time it isn't happening. Do you know why?" The nurse will respond based on which pathophysiologic principle?

"Due to stress, the red blood cells of older adults are not replaced as promptly as younger people."

As part of his diagnostic workup, a 77-year-old male is having his ferritin level analyzed. Which explanation by the health care provider regarding the significance and rationale for this test is most accurate?

"Ferritin is a stored form of iron that indirectly shows me whether you would benefit from iron pills."

A 9 year-old boy has been brought to the emergency department by his father who is concerned by his son's recent fever, stiff neck, pain, and nausea. Examination reveals a petechial rash. Which assessment question by the emergency room physician is most appropriate?

"Has your son had any sinus or ear infections in the last little while?"

The unique clinical presentation of a 3 month-old infant in the emergency department leads the care team to suspect botulism. Which assessment question posed to the parents is likely to be most useful in the differential diagnosis?

"Have you ever given your child any honey or honey-containing products?"

A student states, "It seems like helper T cells do a lot more than just 'help' the cellular immunity process". Which response listed below best conveys an aspect of the role of CD4+ helper T cells in immunity?

"Helper T cells play a major role in stimulating and regulating the whole process."

A 23-year-old man has received a recent diagnosis of appendicitis. The nurse providing care for the man is explaining that the inflammation of his appendix is playing a role in his body's fight against the underlying infectious process. Which teaching points should the nurse eliminate from client education?

"Inflammation helps your body to produce the right antibodies to fight the infection."

A single mother who was raised in a traumatic environment is very protective of her child. She expresses a desire to keep her child from experiencing stressful situations in hopes of promoting healthy development. What response by clinician's is most helpful?

"It is normal to experience stressors periodically and this can enhance healthy development."

As part of her prenatal education, a 29-year-old woman who is pregnant with her first child is receiving teaching from her primary care provider. Which statement by the woman reflects an accurate understanding of HIV transmission?

"It's discouraging to know that my breast milk can pass on HIV to my baby."

A nurse is changing the wound dressing on a coccyx-region pressure injury of an immobilized client. The existing dressing is saturated with both watery, clear discharge and foul, grayish-colored liquid. Which entry in the client's chart most accurately documents these findings?

"Large amounts of suppurative and serous exudates noted."

Which teaching point would be most appropriate with a client who has a recent diagnosis of Von Willebrand disease?

"Make sure that you avoid taking aspirin."

The physician knows the client, G1P0, has correctly understood the prenatal education regarding sexually transmitted infection as evidenced by which statement listed below?

"My baby could become infected either across the placenta or during the birth itself."

An 8-week-old boy has been recently diagnosed with a severe combined immunodeficiency (SCID). His parents have performed a significant amount of research on the Internet and have brought a large amount of material to discuss with their care provider. Which statement best reflects an accurate understanding of their son's health situation?

"Our son likely has a deficiency of B-lymphocytes and can't produce antibodies."

Amniocentesis has suggested that a couple's first child will be born with sickle cell disease. The parents are unfamiliar with the health problem and their caregiver is explaining the complexities. Which statement by the parents would suggest a need for further teaching or clarification?

"Our son or daughter likely won't show the effects of sickling until he or she is school-aged because of the different hemoglobin in babies."

A male client with a history of heavy alcohol use has been admitted to the hospital for malnutrition and suspected pancreatitis. The client's diagnostic workup suggests alcoholic ketoacidosis as a component of his current health problems. He is somewhat familiar with the effect that drinking has had on his nutrition and pancreas, but is wholly unfamiliar with the significance of acid-base balance. How best could his care provider explain the concept to him?

"The chemical processes that take place throughout your body are thrown off very easily when your body is too acidic or not acidic enough.

A 30-year-old woman who has given birth 12 hours prior is displaying signs and symptoms of disseminated intravascular coagulation (DIC). The client's husband is confused as to why a disease of coagulation can result in bleeding. Which statement by the nurse best characterizes DIC?

"So much clotting takes place that there are no available clotting components left, and bleeding ensues."

A client with a diagnosis of hemolytic anemia has gone to a community-based laboratory for follow-up blood work. The lab technician confirms with the client that hematocrit is one of the components of the blood work. The client replies, "I thought the point of the blood work was to see how many red blood cells I have today." How could the technician best respond to the client's statement?

"The hematocrit measures the mass that your red blood cells account for in a quantity of your blood."

The nurse knows which of statement below is appropriate to be included in an education session for a 21-year-old male with a diagnosis of malaria?

"The infectious organisms are considered tiny, single-celled animals, given their complete eukaryotic machinery."

A 40-year-old male client is shocked to receive a diagnosis of mature B-cell lymphoma, and is doing research on his diagnosis on the Internet. Which statement that he reads on various websites is most reliable?

"The lymph nodes are usually affected, and often the spleen and bone marrow."

A female client with a new diagnosis of systemic lupus erythematosus (SLE) has been told that this is an autoimmune disease whereby the immune system is attacking the body's cells and tissue. She knows that she has inflammation and tissue damage. She asks her nurse to explain "What cells in the body are triggering this inflammation?" The nurse responds:

"The lymphocytes that migrate to the brain where they secrete cytokines, which trigger inflammation."

A community health nurse is conducting an immunization clinic when a 14-year-old client asks, "Does this vaccine make me immune to the disease?" What is the nurse's best response?

"The vaccine imitates a disease without causing illness, resulting in the creation of antibodies to the disease."

A potential donor is angry at the personal nature of the questions about HIV risk factors that he is required to answer at a blood collection center, and states that simple blood testing should suffice. How can the lab tech at the center best respond?

"There's a period shortly after someone is infected with HIV when blood tests might still be negative."

After assessment, the nurse asks how long the client has had red, pinpoint hemorrhages on the lower legs. The client responds, "This is the first time I have noticed this. What is wrong with me that is causing these small hemorrhages?" Which response by the nurse is most accurate?

"These hemorrhages are called petechiae and occur when platelets are deficit."

A client has been diagnosed with anemia. The physician suspects an immune hemolytic anemia and orders a Coombs test. The client asks the nurse what this test will tell the doctor. The nurse replies:

"They are looking for the presence of antibody or complement on the surface to the RBC."

A toddler is displaying signs/symptoms of weakness and muscle atrophy. The pediatric neurologist suspects it may be a lower motor neuron disease called spinal muscular atrophy (SMA). The client's family asks how he got this. The clinician will respond:

"This is a degenerative disorder that tends to be inherited as an autosomal recessive trait."

The nurse is caring for a client who has just been diagnosed with shingles. The client states, "I have not been around anyone with this condition. How could I have caught it?" What is the nurse's best response?

"This is a latent virus that you likely were exposed to as a child and it has now become active in the form of shingles."

A child with leukemia hears the health care provider tell the parents that the child has "pancytopenia." This child asks the nurse, "What does 'pancy' mean? Am I going to die?" Which response by the nurse is most appropriate for this 10-year-old child?

"This means the chemotherapy has lowered your blood counts including red blood cells, white blood cells and platelets. You are very sick, but we are working hard to make you healthy again."

Following a motor vehicle accident 3 months prior, a 20-year-old female who has been in a coma since her accident has now had her condition declared a persistent vegetative state. How can her care providers most accurately explain an aspect of her situation to her parents?

"Though she still goes through a cycle of sleeping and waking, her condition is unlikely to change."

A client has been diagnosed with herpes simplex virus. The client states that, "modern medicine produces more and more antivirals every year and so the treatment should be simple." Which statement is the best response?

"Treatment options for viruses are often limited because what destroys viruses often damages your own body cells."

A 70-year-old woman has received a diagnosis of chronic myelogenous leukemia (CML) after a clinical investigation sparked by the presence of leukocytosis in her routine blood work. What clinical course should her care provider tell her to expect?

"You could remain the chronic stage of CML for several years before it accelerates and culminates in a crisis."

A client who is recovering from burn injuries is discussing his prognosis with a physician. Which teaching point about expectations for healing should the physician offer?

"You may find that the scar is a bit smaller than the area of the wound."

A health care professional has recommended biofeedback to a client as a method of dealing with the high levels of stress. Which explanation should the nurse offer the client to explain biofeedback treatment?

"You will be taught how to gain control over skeletal muscle contractions."

The nurse on the cardiac unit has noted that the client's potassium level is 6.1 mEq/L (6.1 mmol/L). The nurse has notified the physician and removed the banana from the client's meal tray. When explaining the nursing actions to the client, which statement is appropriate?

"Your potassium level is high so I need you let me know if you feel numbness, tingling or weakness."

A baseball player was hit in the head with a bat during practice. In the emergency department, the physician tells the family that he has a "coup" injury. How will the nurse explain this to the family so they can understand?

"Your son has a contusion of the brain at the site where the bat hit his head."

The parents of a 3-year-old boy have brought him to a pediatrician for assessment of the boy's late ambulation and frequent falls. Subsequent muscle biopsy has confirmed a diagnosis of Duchenne muscular dystrophy. Which teaching point should the physician include when explaining the child's diagnosis to his parents?

"Your son will be prone to heart problems and decreased lung function because of this."

The nurse is providing care for a client with a diagnosis of amyotrophic lateral sclerosis (ALS). The nurse recognizes that which mechanism is suspected to play a role in the cellular death associated with ALS?

Apoptosis

A client is suspected of having bacterial meningitis. Which action should the nurse complete first?

Administer antibiotics

The clinician is preparing a client for bone marrow transplant. The client asks, "How long will it take for the new marrow to make the blood cells?" What is the best response?

Although it can vary, we will administer medication that will stimulate the bone marrow to create the cells.

A nurse is teaching a group of older adults about the value of including foods containing antioxidants in their diet. Which statement best captures the rationale underlying the nurse's advice?

Antioxidants inhibit the actions of reactive oxygen species.

A clinician is providing care for a 17-year-old boy who has experienced recurrent sinus and chest infections throughout his life and presently has enlarged tonsils and lymph nodes. Blood work indicated normal levels of B cells and free immunoglobins but a lack of differentiation into normal plasma cells. The boy is currently receiving intravenous immunoglobulin (IVIg) therapy. What is the boy's most likely diagnosis?

Common variable immunodeficiency

A woman gives birth to a small infant with a malformed skull. The infant grows abnormally slowly and shows signs of substantial cognitive and intellectual deficits. The child also has facial abnormalities that become more striking as it develops. What might you expect to find in the mother's pregnancy history?

Chronic alcohol use

The nurse is reviewing the treatment plan with parents of a 6-month-old diagnosed with severe β-thalassemia. Which statements by the parents demonstrate they understand the treatment plan? Select all that apply.

a. "Transfusions are needed to prevent skeletal abnormalities from developing." b. "A stem cell transplant may be used as part of the treatment plan."

A parent brings a 12-year-old to the emergency room with multiple bite wounds to the arms and hands from a stray cat. Which interventions should the nurse be prepared to apply? Select all that apply.

a. Administration of prophylactic antibiotics b. Irrigation of the wounds c. Rabies prophylaxis d. A review of the client's immunization record

Which infectious agent would be a site-specific pathogen and not spread throughout the body via the circulatory system? A client diagnosed with:

Helicobacter pylori diagnosed after an endoscopic procedure.

A client is admitted to a rehabilitation center after hospital treatment for an ischemic stroke. What aspects of the client's history are risk factors for ischemic stroke? Select all that apply.

a. Blood pressure historically in the range of 150/90 to 160/100 mm Hg b. Diagnosed with type 2 diabetes 8 years ago c. Black male

Which clients would be at risk for developing nonthrombocytopenic purpura? Select all that apply.

a. Child adopted from India and displaying malaise, lethargy, and petechiae all over the body associated with suspected scurvy. b. 55-year-old client diagnosed with Cushing disease displaying bruises, weight gain with a buffalo hump, and "moon" face

A widow who lost her husband a few weeks ago is having trouble with insomnia. When visiting with her health care provider, the provider suggests a prescription to help her regain a normal circadian pattern. This is based on the fact that interruption of sleep-wake cycles can cause which problems? Select all that apply.

a. Alterations in immune function that can result in an infection b. An increased risk in accidents when sleep deprived, similar to those under the influence of alcohol

A nurse at a long term care facility provides care for an 85 year-old man who has had recent transient ischemic attacks (TIAs). Which statement best identifies future complications associated with TIAs?

TIAs resolve rapidly but may place the client at an increased risk for stroke.

A 12-year-old client who experienced trauma is now having trouble dealing with many forms of conflict in the home and becomes distraught. What is the best approach for the clinician to recommend to the parents?

Talk about the past trauma often and encourage the child to think positively about the future.

A 68-year-old male client with aortic stenosis secondary to calcification of the aortic valve is receiving care. Which statement best captures an aspect of this client's condition?

The client has possibly undergone damage as a result of calcification following cellular injury.

A 56 year-old female hospital client with a history of alcohol abuse is receiving intravenous (IV) phosphate replacement. Which health problem will this IV therapy most likely resolve?

The client is acidotic and has impaired platelet function.

A client has been diagnosed with multiple sclerosis. What should the clinician include in client teaching to help reduce the frequency of exacerbations? Select all that apply.

a. Avoid stressful situations. b. Avoid extremes of environmental temperature. c. Stay up-to-date with your vaccines.

The nurse is caring for a teenager with sickle cell anemia. When comparing new laboratory results with the client's baseline, which results should the nurse prioritize as requiring intervention? Select all that apply.

a. Elevated white blood cell count b. Elevated serum osmolality c. Low pH on an arterial blood gas

The nurse is caring for a client with multiple myeloma. What aspects of care should the nurse prioritize? Select all that apply.

a. Encourage increased fluid intake b. Monitor urine output c. Assist for ambulation

Two years after chemotherapy and radiation therapy for lung cancer, a 72-year-old client reports being extremely tired all the time. The physician suspects the client may have developed aplastic anemia. The nurse assessing the client will likely find which clinical manifestation of aplastic anemia? Select all that apply.

a. Excess bleeding from gums and nose b. Small spots of skin hemorrhages over entire body c. Complaints of weakness and fatigue

The nurse is caring for a client with thrombotic thrombocytopenic purpura (TTP) and notices a yellow discoloration of the client's sclera. Which are likely possible causes of this finding? Select all that apply.

a. Hypoperfusion of the liver b. Accelerated intravascular hemolysis

While volunteering in an HIV clinic in a big city, the clinician notices a new mom and her 6-month old child in the waiting room. Upon assessing the infant for possible HIV infection, the clinician will be assessing for which clinical manifestations of HIV infection? Select all that apply.

a. Lack the coordination to play with toys/stuffed animals. b. Listlessness and poor eye contact. c. History of repeated episodes of bacterial pneumonia and ear infections. d. Weighing him to determine if he is gaining 1.5 to 2 pounds/month.

A client with severe anemia asks, "What happens if my bone marrow cannot produce sufficient numbers of red blood cells due to my bleeding?" Which response(s) by the nurse correctly identifies the organs that can resume hematopoiesis? Select all that apply.

a. Liver b. Spleen

From the current health histories, which client(s) is at risk for developing secondary forms of immune thrombocytopenic purpura (ITP)? Select all that apply.

a. Middle-aged client scheduled for gastroscopy to confirm Helicobactor pylori b. Former IV drug user recently diagnosed with hepatitis C virus

The clinician suspects a spinal cord injury client is developing autonomic dysreflexia. Which assessment findings would confirm the development of this complication? Select all that apply.

a. Pulse rate 49 b. Complains of a pounding headache c. BP 180/98

The nurse is caring for neonates undergoing treatment for hyperbilirubinemia. Which assessment findings should the nurse prioritize? Select all that apply.

a. Reduction in feeding b. Rigidity c. Lethargy

A patient is brought to the emergency department for an overdose of aspirin. Clinicians caring for this client should anticipate which clinical manifestations? Select all that apply.

a. Respiratory rate of 40 b. ABG report: pH 7.50, PCO2 31 mmHg (4.12 kPa), HCO3 level 19 mmol/L

A physician is caring for a client with posttraumatic stress distress disorder (PTSD). Which behaviors would the physician expect the client to manifest? Select all that apply.

a. Sleep disturbance with vivid nightmares b. Reluctance to participate in group discussions c. Inability to recall the traumatic events

A young client has just been diagnosed with xeroderma pigmentosum. When teaching this family about this disease, the nurse should emphasize which teaching points? Select all that apply.

a. The best time to allow the child to play outside is in the evening hours after the sun goes down. b. Wear long sleeves, long pants, gloves, a hat, sunglasses with side shields, and sunscreen while outdoors.

The health care provider is ordering a complete blood count (CBC). What information should the nurse expect to be able to gather from the results? Select all that apply.

a. The presence of leukocytosis b. Presence of thrombocytopenia c. If the client has anemia

The nurse is caring for a client with sepsis who has developed disseminated intravascular coagulation (DIC). Which assessments should the nurse prioritize? Select all that apply.

a. Urine output b. Level of consciousness c. Oxygen saturation

Following head injury from a fall, the child's CT scan reveals a moderate brain injury contusion. Which manifestations will the nurse more than likely assess on this child that supports this diagnosis? Select all that apply.

a. Weakness or slight paralysis affecting one side of the body b. Periods of unconsciousness c. Aphasia at times

The nurse is caring for a client who is suspected to have multiple myeloma. What findings will the nurse use as evidence the client has multiple myeloma? Select all that apply.

a. X-rays demonstrate bone loss b. Bence Jones proteins in urine c. 20% plasma cells in bone marrow biopsy

The family member of a client with terminal metastatic cancer who is experiencing cachexia-related weight loss asks the nurse why the client is losing weight despite taking in a large amount of calories per day. What is the nurse's best response?

"This weight loss is related to the cancer itself and occurs despite an intake of adequate calories."

A 60-year-old man has presented to a clinic and is requesting screening for tumor markers after reading about them in a magazine. What can the clinician most accurately tell the man about the clinical use of tumor markers?

"Tumor markers alone aren't enough to confirm whether you have cancer or not, so they're not a very useful screening tool."

A member of the health care team is researching the etiology and pathogenesis of a number of clients who are under his care in a hospital context. Which aspect of clients' situations best characterizes pathogenesis rather than etiology?

A client who is has increasing serum ammonia levels due to liver cirrhosis.

A clinician who works on a cardiac care unit of a hospital is providing care for a number of clients. Which client most likely has a genetic disorder arising from inheritance of a single gene?

A tall, thin, myopic, 28-year old woman with mitral valve prolapse

A worker in a warehouse is trying to have children but think he or she has handled "mercury" while cleaning equipment. Which statement by the occupational nurse is most appropriate at this time?

Most mercury toxicity involves central nervous system changes.

The family of a 68-year-old man who is in the end stages of small-cell lung cancer are distraught at his visible body wasting that has worsened in recent weeks. Which phenomena best accounts for the client's anorexia and cachexia?

Products of the tumor itself as well as a hypermetabolic state cause cachexia.

A 77-year-old man is a hospital inpatient admitted for exacerbation of his chronic obstructive pulmonary disease (COPD), and a respiratory therapist (RT) is assessing the client for the first time. Which aspect of the client's current state of health would be best characterized as a symptom rather than a sign?

The client notes that he has increased work of breathing when lying supine.

While traveling throughout Asia, a young couple was exposed to rubella. During their first clinic visit, the couple found out they were pregnant and express concern about their possible exposure to rubella. The nurse knows that this infant is at high risk for which possible complications related to rubella exposure? Select all that apply.

a. Blindness or cataracts b. Deafness

The nurse is caring for a client who was just told she is 4 months pregnant and was unaware of the pregnancy. The nurse is reviewing any medications that the client may have taken in the past 5 months. Which characteristic of the medication(s) will the nurse assign the most significance when considering if the medication may be teratogenic?

Whether the medication is lipid or water-soluble

Laboratory testing is ordered for a male client during a clinic visit for routine follow-up assessment of hypertension. When interpreting lab values, the nurse knows:

a normal value represents the test results that fall within the bell curve.

During a discussion with a client recently diagnosed with stage IV kidney cancer, which statement is most accurate?

"Many times, kidney cancer goes completely undetected until a metastatic lesion is found in the lung."

Two health care workers are comparing the etiology and incidence of multifactorial inheritance disorders and single-gene disorders. Which statement best captures the relationship between the two types of genetic disorders?

"A couple with a child with a multifactorial disorder have a higher risk of having another with the same disorder."

A new older female client at a long term care facility has a diagnosis of neurofibromatosis type 1. As part of the intake assessment protocol for the facility, the clinical educator is teaching the care staff about the diagnosis. Which statement most accurately conveys an aspect of neurofibromatosis?

"Her diagnosis puts her at higher risk of developing a malignant neoplasm."

A male client has an autosomal dominant disorder. The client and his partner are considering starting a family. Which statement indicates the client has an adequate understanding of the genetic basis of this health problem?

"I know that a single mutant allele is to blame for the health problem."

A client who has a diagnosis of lung cancer is scheduled to begin radiation treatment. The nurse knows that which statement about potential risks of radiation is most accurate?

"Some clients experience longer-term irritation of skin adjacent to the treatment site."

A new client who suffered a myocardial infarction requires angioplasty and stent placement. He has arrived to his first cardiac rehabilitation appointment. In this first session, a review of the pathogenesis of coronary artery disease is addressed. Which statement by the client verifies to the nurse that he has understood the nurse's teachings about coronary artery disease?

"Sounds like this began because of inflammation inside my artery that made it easy to form fatty streaks which led to my clogged artery."

A couple who are pregnant with their first child have made an appointment with a clinical geneticist to discuss prenatal screening. The man states that they, "just want to make sure that there is nothing wrong with our baby." How could the clinician best respond to this statement?

"Testing the umbilical blood and performing amniocentesis can give us some information, but not a guarantee."

A client overheard the provider discussing the case and mentioning a "mutation in the TP53 gene." The client asks the nurse, "What does that mean?" Which response is the most appropriate for this client?

"The TP53 gene is a group of tumor suppressor genes associated with lung, breast and colon cancer."

As part of a first aid class, a health care instructor is teaching a group of industrial workers about how electrical injuries can cause cell damage. Which statement made by one of the workers indicates that further teaching is necessary?

"The greater the skin resistance, the greater the amount of deep and systemic damage a victim is likely to incur."

The nurse is teaching a group of new mothers about postpartum nutrition, when one of the clients states that she was told to avoid eating fish too often due to the risk of mercury poisoning. Which response by the nurse most accurately addresses the clients concerns?

"There are some modest risks, but they are only associated with some long-living fish like tuna."

As part of her prenatal care, a pregnant woman and her partner are being taught by a community health nurse. Which point about the teratogenic effects of different substances should the nurse include in teaching?

"Your developing baby is most vulnerable during the first 2 months of your pregnancy."

Of the following situations, which one would be an example of a maladaptive cellular change?

44-year-old male with a 60 pack/year smoking history who was diagnosed with a histological grade-3 lung cancer

Which client is at high risk for developing dilated cardiomyopathy?

44-year-old noncompliant female who forgets to take her hypertensive medications

A 41-year-old female with a family history has had a baseline mammogram. She states that she performs monthly self breast exams but really has a hard time evaluating her lumps since she has numerous cysts. At her annual mammogram, the technician views a suspicious area and refers her to the radiologist. She asks the nurse in the office, "How can a lump appear so quickly?" The nurse's response is based on which principle?

A tumor is undetectable until it has doubled 30 times and contains at least 1 billion cells.

Which pregnant woman has most likely encountered the greatest increase in the risk that her child will have a fetal anomaly?

A woman who has herpes simplex and who has recently recovered from endocarditis.

As part of a screening program for prostate cancer, men at a senior citizens center are having their blood levels of prostate-specific antigen (PSA) measured. Which statement would best characterize high positive predictive value but low negative predictive value for this screening test?

All of the men who had high PSA levels developed prostate cancer; several men who had low PSA levels also developed prostate cancer.

A nurse is triaging clients at a disaster site. Local facilities have different specialized units. To what facility should the nurse send a client who has sustained an electrical injury to his left thigh?

Burn unit

An epidemiologist is conducting a program of research aimed at identifying factors associated with incidence and prevalence of congenital cardiac defects in infants. The researcher has recruited a large number of mothers whose infants were born with cardiac defects as well as mothers whose infants were born with healthy hearts. The researcher is comparing the nutritional habits of all the mothers while their babies were in utero. Which type of study is the epidemiologist most likely conducting?

Case-control study

The nurse is questioning the validity of oxygen saturation readings from the new oximeters on the unit. Which action should the nurse take to best determine if the results from the oximeter are valid?

Compare clients' results to measurements taken using arterial blood gas analyses.

A physician is working with a 30-year-old male client with Down syndrome who has been admitted to hospital with a diagnosis of acute leukemia. Which physical assessment finding would the physician be more likely to find in an examination of this client than other clients without Down syndrome?

Congenital heart defects

A nurse in the emergency department admits a male client who has experienced severe frostbite to his hands and toes after becoming lost on a ski hill. The nurse recognizes that which phenomena has contributed to his tissue damage?

Decreased blood flow has induced hypoxia.

An 11-year-old girl is suspected of having Turner syndrome. Which diagnostic test would be the most useful component of screening to confirm or rule out the diagnosis?

Echocardiogram

Unbeknownst to her or her care team, a 51-year-old woman's breast cancer has an etiology rooted in the fact that tumor suppressing genes are present but have been silenced. Consequently, she has not synthesized normal cancer-suppressing proteins and neoplasia has resulted. What process has accounted for the woman's cancer?

Epigenetic mechanisms

Parents bring their 18-month-old child to the emergency room exhibiting behavior changes and vomiting. They are concerned the child ingested something in the older home they are renovating. Laboratory findings indicate low hemoglobin and elevated creatinine. Which diagnostic test should the nurse advocate for first?

Erythrocyte protoporphyrin (EP) level

A 26-year-old man who survived childhood acute lymphocytic leukemia (ALL), one of the most common childhood cancers, now complains of weakness, fatigue, and shortness of breath. His treatment for ALL likely included anthracyclines. What is the most likely cause of his symptoms?

Heart failure resulting from childhood chemotherapy

The laboratory technologists are a discussing a new blood test that helps establish a differential diagnosis between shortness of breath with a cardiac etiology and shortness of breath with a respiratory/pulmonary etiology. A positive result is purported to indicate a cardiac etiology. The marketers of the test report that 99.8% of clients who have confirmed cardiac etiologies test positive in the test. However, 1.3% of clients who do not have cardiac etiologies for their shortness of breath also test positive. Which statement best characterizes this blood test?

High sensitivity, low specificity

A community health nurse is teaching a group of recent graduates about the large variety of factors that influence an individual's health or lack thereof. The nurse is referring to the Healthy People 2020 report from the U.S. Department of Health and Human Services as a teaching example. Of the following aspects discussed, which would be considered a determinant of health that is outside the focus of this report?

The client has a family history of cardiovascular disease related to hypercholesterolemia and remains noncompliant with the treatment regime.

An oncology nurse who has worked for many years providing care for children with cancer has taken a job on an adult oncology unit of a hospital. What differences might the nurse anticipate in this new job?

There will be a greater number of cancers that are epithelial in origin.

The nurse is planning care for a group of clients. Which clients should the nurse prioritize as "at risk" for hypercalcemia and advocate for monitoring calcium levels? Select all that apply.

a. A client who has breast cancer with bone metastasis b. A client with prolonged immobility c. A client with hypophosphatemia

The neuroscience nursing unit has developed a set of step-by-step directions of what should occur if a nursing assessment reveals the client may be exhibiting clinical manifestations of a cerebrovascular accident (CVA). Which statement about clinical practice guidelines are accurate? Select all that apply.

a. A meta-analysis could be utilized to combine evidence from different studies to produce a more accurate diagnostic method. b. When developing a CVA set of step-by-step directions, the nursing unit should ask for assistance from experts in the neuroscience field. The potential users of the guidelines should pilot test it for further feedback. c. The development of evidence-based practice guidelines require a research review from different studies to develop the most accurate diagnostic method to implement.

A pregnant client is exploring options related to prenatal testing due to a history of inherited disorders. What information should the nurse include when informing the client about various prenatal tests? Select all that apply.

a. All of the tests used to examine fetal DNA are invasive and carry associated risks. b. An ultrasound will be required for either amniocentesis or chorionic villus sampling.

The nurse is planning care for a client who is receiving antimetabolite chemotherapy. What should the nurse include in the plan of care? Select all that apply.

a. Daily weights b. Falls prevention protocol c. Monitor bowel movements closely.

A pregnant client learns she has fragile X syndrome through prenatal screening and asks how this might affect the unborn child. What information should the nurse include in the response? Select all that apply.

a. Daughters are less severely affected than sons by fragile X syndrome. b. You have a 50% chance of passing the condition to a child of either sex. c. Males are less likely to be born with fragile X syndrome.

Which situation would be classified as a complication of a disease or outcome from the treatment regimen? Select all that apply.

a. Development of pulmonary fibrosis following treatment with bleomycin, an antibiotic chemotherapy agent used in treatment of lymphoma b. Massive pulmonary emboli following diagnosis of new onset atrial fibrillation.

The nurse is caring for a client with arterial insufficiency of the left leg with gangrenous wounds on the second and third toes. What characteristics of the wounds should the nurse evaluate as expected? Select all that apply.

a. Diminished pulse strength b. Darkened appearance of affected tissue c. Atrophy to the affected toes d. A clear demarcation between the healthy and affected tissue

A male child was born with Klinefelter syndrome. As an adolescent, the nurse will assess which clinical manifestations of Klinefelter syndrome? Select all that apply.

a. Enlarged breast tissue b. Tall stature out of proportion c. Sparse facial and pubic hair

While attending an international nursing conference, many discussions and break-out sessions focused on the World Health Organization's (WHO) views on health. Of the following comments made by nurses during a discussion session, which statements would be considered a good representation of the WHO definition? Select all that apply.

a. Interventions geared toward keeping the older adult population diagnosed with diabetes mellitus under tight blood glucose control by providing in-home cooking classes b. Increase in the number of chair aerobics classes provided in the skilled care facilities c. Interests in keeping the older adult population engaged in such activities as book reviews and word games during social time d. Providing handwashing teaching sessions to a group of young children

The mother of an 18-month-old child is concerned that her child is lethargic and not eating foods that he normally enjoys. She takes him to the pediatrician for a check-up. Which clinical manifestations lead the health care provider to suspect the child may have a neuroblastoma? Select all that apply.

a. Large, protruding abdomen b. Weight loss c. Crying when joints/position changed

The health care provider is discussing the treatment protocol using ionizing radiation to treat the client's cancer. The client asks, "What side effects can occur with this treatment?" Which response(s) is accurate? Select all that apply.

a. Low blood counts b. Excessive diarrhea

Blood-borne cancerous cells have recently spread from a woman's primary pancreas tumor to her bones. Which components of the woman's immune system are likely to be directly involved in the attempt to eradicate the potential metastasis? Select all that apply.

a. Macrophages b. T lymphocytes c. Natural killer (NK) cells d. B lymphocyte cells

The nurse is caring for a client with small cell lung carcinoma who has developed syndrome of inappropriate antidiuretic hormone (SIADH). What should the nurse incorporate into the plan of care? Select all that apply.

a. Maintenance of fluid restriction b. Daily weights c. Frequent repositioning

While preparing a client about to undergo percutaneous umbilical cord blood sampling (PUBS), which educational information should the nurse provide prior to the procedure? Select all that apply.

a. Once the procedure is begun, you must lie very still since they will be inserting a needle through the uterine wall. b. During the procedure, an ultrasound will be utilized to guide the catheter into the correct position.

The nurse is caring for a 45-year-old client undergoing radiotherapy of the mediastinal nodes due to lymphoma. Which interventions should the nurse prioritize? Select all that apply.

a. Performing respiratory assessment b. Avoiding invasive procedures c. Assessing for fatigue

The nurse working in a pediatric office is scheduled to assess a female adolescent diagnosed with neurofibromatosis (NF) Type 1. During this assessment, the nurse should be assessing the teenager for which clinical manifestations of NF-1? Select all that apply.

a. Speech impediments b. Reports of having a hard time concentrating in school c. Severe scoliosis

While taking a prenatal history, the nurse would be most concerned about severe teratogenic effects on the fetus if the mother admits to taking which medication prior to finding out that she was pregnant? Select all that apply.

a. Tetracycline for acne b. Warfarin for chronic atrial fibrillation c. Ethyl alcohol ingestion regularly several times per week d. Isotretinoin for acne

A nurse is investigating the epidemiologic factors influencing breast cancer for women in a population. What information should the nurse include? Select all that apply.

a. The age of women at the time of diagnosis b. The geographic location of women diagnosed with breast cancer

When educating a client who has tested positive for human T-cell leukemia virus-1 (HTLV-1), what mode(s) of transmission should the nurse discuss to prevent the spread? Select all tht apply.

a. blood by sharing needles b. infants through breast milk c. sexual intercourse

A community health nurse is discussing preventative vaccines with a group of primigravda women. Which cancer-causing virus(es) should the nurse discuss that have a vaccine to prevent developing the disease? Select all that apply,

a. human papillomavirus (HPV) b. hepatitis B

A home health nurse is making a visit to a family with an 8-month-old infant with severe motor deterioration. The physician has diagnosed the infant with Tay-Sachs disease. The parents are asking the nurse why this happened. The nurse will base her answer knowing the root cause of Tay-Sachs is:

an enzyme defect causing abnormal lipid accumulation in the brain.

A nurse is considering setting up a screening program for a specific health condition in a population. What characteristic of the condition would need to be true for the nurse to justify screening a population? The condition should:

be asymptomatic at an early stage.

The nurse is caring for a client with Marfan syndrome. Which assessment finding should the nurse prioritize as requiring the most emergent intervention? A sudden:

drop in blood pressure from 130/90 to 95/50 mm Hg.

The nurse is teaching new nursing assistants on the unit about the phenomenon of muscle hypertrophy. Which client on the unit is most likely to experience muscle hypertrophy? A client with:

hypertension, obesity, and decreased activity tolerance.

A client has stage IV cancer but further testing is needed to determine the site of origin of this metastatic tumor. Which form of testing will the health care provider be discussing with this client?

immunohistochemistry

The nurse is educating a client who is undergoing gamma knife radiosurgery for a brain metastasis. What advantage to this procedure should the nurse share when comparing it to other conventional treatments?

less cognitive dysfunction

As of Nov. 1, 2012, there were a total of 10 confirmed cases of hantavirus infection in people who were recent visitors (mid-June to end of August, 2012) to Yosemite National Park. Three visitors with confirmed cases died. Health officials believe that 9 out of the 10 cases of hantavirus were exposed while staying in Curry Village in the Signature Tent Cabins. This is an example of:

the incidence of people who are at risk for developing hantavirus while staying in Yosemite National Park.

A male international business traveler has returned from a trip to Indonesia. While there, he hired a prostitute for companionship and engaged in unprotected sex on more than one occasion. Unbeknownst to him, this prostitute harbored the hepatitis C virus. Upon return to the U.S., he exhibited no symptoms and returned to his usual activities. During this period of no outward symptoms, the disease would be classified as being in:

the preclinical stage of disease.


Ensembles d'études connexes

Chapter 7 Inquizitive Texas Government

View Set

psych test 7, Psychology test 1, Psychology test 4, Psychology test 5, Psychology Test 3, Psychology test 2, Psych test 6

View Set

nclex GU, Pediatric GU questions Nclex, renal gu nclex, Renal & GU- NCLEX, GU NCLEX 3500, NCLEX GU

View Set

Chapter 23: Modern Industry and Mass Politics, 1870-1914

View Set

Chronic Care Lab/Clinical Dosage Calc Study Guide

View Set

Leadership ch 7: Nursing Leadership strategic and operational planning

View Set

Chapter 6 Cities, Immigrants, and Farmers

View Set

RELIGION: Who Am I: Genesis Patriarch Stories (Abraham-Jacob)

View Set